Difference between revisions of "2018 AMC 12B Problems/Problem 12"

(Changed the variable definition so it is more manageable. Also, used the ordered list command for the three inequalities.)
(Solution)
 
(2 intermediate revisions by one other user not shown)
Line 14: Line 14:
 
Recall that <math>x>0.</math> We apply the Triangle Inequality to <math>\triangle ABC:</math>
 
Recall that <math>x>0.</math> We apply the Triangle Inequality to <math>\triangle ABC:</math>
 
<ol style="margin-left: 1.5em;">
 
<ol style="margin-left: 1.5em;">
   <li><math>AC+BC>AB</math> <p>
+
   <li><math>AC+BC>AB \iff x+\left(\frac{30}{x}+3\right)>10</math> <p>
We get
+
We simplify and complete the square to get <math>\left(x-\frac72\right)^2+\frac{71}{4}>0,</math> from which <math>x>0.</math>
<cmath>\begin{align*}
 
x+\left(\frac{30}{x}+3\right)&>10 \\
 
x-7+\frac{30}{x}&>0 \\
 
x^2-7x+30&>0 \\
 
\left(x-\frac72\right)^2+\frac{71}{4}&>0 \\
 
x&>0.
 
\end{align*}</cmath>
 
 
</li><p>
 
</li><p>
   <li><math>AB+BC>AC</math> <p>
+
   <li><math>AB+BC>AC \iff 10+\left(\frac{30}{x}+3\right)>x</math> <p>
We get
+
We simplify and factor to get <math>(x+2)(x-15)<0,</math> from which <math>0<x<15.</math>
<cmath>\begin{align*}
 
10+\left(\frac{30}{x}+3\right)&>x \\
 
x-13-\frac{30}{x}&<0 \\
 
x^2-13x-30&<0 \\
 
(x+2)(x-15)&<0 \\
 
0<x&<15.
 
\end{align*}</cmath>
 
 
</li><p>
 
</li><p>
   <li><math>AB+AC>BC</math> <p>
+
   <li><math>AB+AC>BC \iff 10+x>\frac{30}{x}+3</math> <p>
We get
+
We simplify and factor to get <math>(x+10)(x-3)>0,</math> from which <math>x>3.</math>
<cmath>\begin{align*}
 
10+x&>\frac{30}{x}+3 \\
 
x+7-\frac{30}{x}&>0 \\
 
x^2+7x-30&>0 \\
 
(x+10)(x-3)&>0 \\
 
x&>3.
 
\end{align*}</cmath>
 
 
</li><p>
 
</li><p>
 
</ol>
 
</ol>
 
Taking the intersection of the solutions gives <cmath>(m,n)=(0,\infty)\cap(0,15)\cap(3,\infty)=(3,15),</cmath> so the answer is <math>m+n=\boxed{\textbf{(C) }18}.</math>
 
Taking the intersection of the solutions gives <cmath>(m,n)=(0,\infty)\cap(0,15)\cap(3,\infty)=(3,15),</cmath> so the answer is <math>m+n=\boxed{\textbf{(C) }18}.</math>
  
~MRENTHUSIASM
+
~quinnanyc ~MRENTHUSIASM
 +
 
 +
==Video Solution (HOW TO THINK CRITICALLY!!!)==
 +
https://youtu.be/OfnHE-KxZJI
 +
 
 +
~Education, the Study of Everything
  
 
==See Also==
 
==See Also==

Latest revision as of 13:25, 29 May 2023

Problem

Side $\overline{AB}$ of $\triangle ABC$ has length $10$. The bisector of angle $A$ meets $\overline{BC}$ at $D$, and $CD = 3$. The set of all possible values of $AC$ is an open interval $(m,n)$. What is $m+n$?

$\textbf{(A) }16 \qquad \textbf{(B) }17 \qquad \textbf{(C) }18 \qquad \textbf{(D) }19 \qquad \textbf{(E) }20 \qquad$

Solution

Let $AC=x.$ By Angle Bisector Theorem, we have $\frac{AB}{AC}=\frac{BD}{CD},$ from which $BD=CD\cdot\frac{AB}{AC}=\frac{30}{x}.$

Recall that $x>0.$ We apply the Triangle Inequality to $\triangle ABC:$

  1. $AC+BC>AB \iff x+\left(\frac{30}{x}+3\right)>10$

    We simplify and complete the square to get $\left(x-\frac72\right)^2+\frac{71}{4}>0,$ from which $x>0.$

  2. $AB+BC>AC \iff 10+\left(\frac{30}{x}+3\right)>x$

    We simplify and factor to get $(x+2)(x-15)<0,$ from which $0<x<15.$

  3. $AB+AC>BC \iff 10+x>\frac{30}{x}+3$

    We simplify and factor to get $(x+10)(x-3)>0,$ from which $x>3.$

Taking the intersection of the solutions gives \[(m,n)=(0,\infty)\cap(0,15)\cap(3,\infty)=(3,15),\] so the answer is $m+n=\boxed{\textbf{(C) }18}.$

~quinnanyc ~MRENTHUSIASM

Video Solution (HOW TO THINK CRITICALLY!!!)

https://youtu.be/OfnHE-KxZJI

~Education, the Study of Everything

See Also

2018 AMC 12B (ProblemsAnswer KeyResources)
Preceded by
Problem 11
Followed by
Problem 13
1 2 3 4 5 6 7 8 9 10 11 12 13 14 15 16 17 18 19 20 21 22 23 24 25
All AMC 12 Problems and Solutions

The problems on this page are copyrighted by the Mathematical Association of America's American Mathematics Competitions. AMC logo.png